I would like to evaluate the following integral:
##\displaystyle{\int_{-\infty}^{\infty} dp^{0}\ \delta(p^{2}-m^{2})\ \theta(p^{0})}##
##\displaystyle{= \int_{-\infty}^{\infty} dp^{0}\ \delta[(p^{0})^{2}-\omega^{2}]\ \theta(p^{0})}##
##\displaystyle{= \int_{-\infty}^{\infty} dp^{0}\...
Homework Statement
Find Req in the given circuit
Homework Equations
Series: req = r1+r2...
parallel: 1/req = 1/r1 + 1/r2 ...
The Attempt at a Solution
Without redrawing the circuit I do:
12||60 = 10
10+20 = 30
30 || 30 = 15
15+10 = 25
25||25 = 12.5
12.5+15+5 = 32.5 = Req
This is the correct...
Note from mentor: this thread was originally posted in a non-homework forum, therefore it lacks the homework template.
I was wondering if the electric charge density ##\rho({\bf{r}})## of a point charge ##q## at position ##{\bf{r}}_{0}## is given by...
Homework Statement
Lauryl alcohol is obtained from coconut oil and is used to make detergents. A solution of 5.00 gram of lauryl alcohol in 0.100Kg of benzene freezes at 4.1 degrees Celsius. What is the molar mass of the lauryl alcohol?
Kf for benzene = 5.12, Freezing point of benzene is 5.5...
Dear PF members.
I have trouble understanding how zero sequence currents circulate in a delta winding. For example zero sequence currents such as 3rd harmonics, they will circulate in the delta winding and get trapped, why?
Why can they not go further?
I have recently started to study...
Hi PF!
As outlined in my book ##\delta_{ij} \delta_{jk} = \delta_{ik}## but don't we sum over repeated indices (and the ##j## is repeated)? Can someone explain why we do not sum in this situation?
Thanks!
Homework Statement
hi
i have to find the result of
##\int_{0}^{\pi}[\delta(cos\theta-1)+ \delta(cos\theta+1)]sin\theta d\theta##
Homework Equations
i know from Dirac Delta Function that
##\int \delta(x-a)dx=1##
if the region includes x=a and zero otherwise
The Attempt at a Solution
first i...
The exercises in my imaginary textbook are giving me an ε, say .001, & are making me find a delta, such that all values of x fall within that ε range of .001. The section that I'm working on is called "proving limits." Well, that is not proving a limit. All that's doing is finding values of...
Dear all,
I have a quick question, is the following statement true?
$$\nabla_\textbf{x'} \delta(\textbf{x}-\textbf{x'}) = \nabla_\textbf{x} \delta(\textbf{x}-\textbf{x'})?$$
I thought I have seen this somewhere before, but I could not remember where and why.
I know the identity ##d/dx...
Hey all,
i've found the following expression:
How do they get that? They somehow used the kronecker delta Sum_k exp(i k (m-n))=delta_mn. But in the expression above, they're summing over i and not over r_i??
Best
Hello,
I'm stuck with this exercise, so I hope anyone can help me.
It is to prove, that the density of states of an unknown, quantum mechanical Hamiltonian ##\mathcal{H}##, which is defined by
$$\Omega(E)=\mathrm{Tr}\left[\delta(E1\!\!1-\boldsymbol{H})\right]$$
is also representable as...
Homework Statement
Hey guys. I am having a little trouble answering this question. I am teaching myself calc 3 and am a little confused here (and thus can't ask a teacher). I need to find the limit as (x,y) approaches (0,1) of f(x,y) when f(x,y)=(xy-x)/(x^2+y^2-2y+1).
Homework Equations...
Homework Statement
Homework Equations
Wye:
V_Line = sqrt(3) x V_Phase
Delta:
V_Line = V_Phase
The Attempt at a Solution
My answer doesn't quite match up with the solutions in the book, and was wondering what I went wrong here? Any help is greatly appreciated!
Homework Statement
δ(z*-z0*)δ(z+z0)=?
δ(z*+z0*)δ(z-z0)=?
where 'z' is a complex variable 'z0' is a complex number
Formula is just enough, derivation is not needed.
Homework Statement
What is the product of two Dirac delta functions
δ(Real(z-c))δ(Img(z-c))=?
'z' and 'c' are complex numbers.
This is not a problem, But I just need to use this formula in a derivation that I am currently doing. I just want the product of these two Dirac delta functions as a...
Homework Statement
Differential equation: ##Ay''+By'+Cy=f(t)## with ##y_{0}=y'_{0}=0##
Write the solution as a convolution (##a \neq b##). Let ##f(t)= n## for ##t_{0} < t < t_{0}+\frac{1}{n}##. Find y and then let ##n \rightarrow \infty##.
Then solve the differential equation with...
Can somebody help me? I am studying Faddeev-Popov trick, following the Peskin and Schroeder's QFT book, but I can't understand one thing. After they inserted the Faddeev-Popov identity,
$$I = \int {{\cal D}\alpha \left( x \right)\delta \left( {G\left( {{A^\alpha }} \right)} \right)\det \left(...
In lectures, I have learned that F(k)= \int_{-\infty}^{\infty} e^{-ikx}f(x)dx where F(k) is the Fourier transform of f(x) and the inverse Fourier transform is f(x)= \frac{1}{2\pi}\int_{-\infty}^{\infty} e^{ikx}f(k)dk .
But on the same chapter in the lecture notes, there is an example solving...
Homework Statement
Find the Fourier spectrum of the following equation
Homework Equations
##F(\omega)=\pi[\delta(\omega - \omega _0)+\delta(\omega +\omega_0)]##
The Attempt at a Solution
Would the Fourier spectrum just be two spikes at ##+\omega _0## and ##-\omega _0## which go up to infinity?
Given the definition:
δ(x) = 0 for all x ≠ 0
∞ for x = 0
∫-∞∞δ(x)dx = 1
I don't understand how the integral can equal unity. The integral from -∞ to zero is zero, and the integral from 0 to ∞...
Homework Statement
I am trying to determine whether
$$f(x)g(x')\delta (x-x') = f(x)g(x)\delta (x-x') = f(x')g(x')\delta(x-x')$$
where \delta(x-x') is the Dirac delta function and f,g are some arbitrary (reasonably nice?) functions.
Homework Equations
The defining equation of a delta function...
Homework Statement
[/B]
A very thin plastic ring (radius R) has a constant linear charge density, and total charge Q. The ring spins at angular velocity \omega about its center (which is the origin). What is the current I, in terms of given quantities? What is the volume current density J in...
Homework Statement
Consider the double well potential with two wells separated by delta function in middle.
V(x) = V0 > 0 for x<-a and x>a
0 for -a<x<0 and 0<x<a
αδ for x =0
1. Find Bound state energies
2. Find odd solns and their eigenvalue equation. Give solns in...
Homework Statement
Consider a one-dimensional time-independent Schrodinger equation for an electron in a double quantum well separated by an additional barrier. The potential is modeled by:
V (x) = -γδ(x - a) -γδ(x + a) + βδ(x)
Find algebraic equations which determine the energies (or...
We know that the solutions of time-independent Dirac delta potential well contain bound and scattering states:
$$\psi_b(x)=\frac{\sqrt{mu}}{\hbar}e^{-\frac{mu|x|}{\hbar^2}}\text{ with energy }E_b=-\frac{mu^2}{2\hbar^2}$$
and
$$
\psi_k(x)=
\begin{cases}
A(e^{ikx}+\frac{i\beta}{1-i\beta}e^{-ikx})...
Homework Statement
For each of these sketch and provide a formula for the function (i.e. in terms of ##u(t)##, ##\delta(t)##) and its derivative and anti-derivative. Denote the ##\delta## function with a vertical arrow of length 1.
(a) ##f(t)=\frac{|t|}{t}##
(b) ##f(t)=u(t) exp(-t)##...
Homework Statement
I need to integrate
##\frac{A}{2a\sqrt{2\pi}} \int_{-\infty}^{\infty} \frac{e^{ik(x+x')}}{(b^2+k^2)}dk##
I have tried substitution and integration by parts and that hasn't worked. I can see that part of it is the delta function, but I don't really know how to use that fact...
Homework Statement
I have a particle which is initially in a bound state for a given voltage in the form of a delta function at the origin,
V = -αδ(x)
initial state is ψα = (√αm)/h2*exp(-m*α*|x|/(h2)
At t=0, voltage is changed to V = -βδ(x)
both α and β are greater than zero. Right now I'm...
Homework Statement
Find the solution to:
$$\frac{d^2}{dt^2} x + \omega^2 x = \delta (t)$$
Given the initial condition that ##x=0## for ##t<0##. First find the general solution to ##t>0## and ##t<0##.
Homework Equations
The Attempt at a Solution
This looks like a non-homogeneous second...
Hi, in the book 'Introduction to Quantum Mechanics' by Griffiths, on page 71 in the section 'The Delta-Function Potential' he states that the general solution to time independent Schrodinger Equation is $$\psi(x) = Ae^{-\kappa x} + B e^{\kappa x}$$
he then notes that the first term blows up as...
Using the Delta Netowrk from this image: http://www.belden.com/images/B23_WyevsDelta.jpg
Between Y-X, there should be 120V.
Between Y-Z, there should be 120V.
Between X-Z, there should be 120V.
But wait!
If we assign values to make the above statements true...
Call X = 0V, and Y = 120V, so...
Hi, friends! I have been able to understand, thanks to Hawkeye18, whom I thank again, that, if ##\mathbf{J}## is measurable according to the usual ##\mathbb{R}^3## Lebesgue measure ##\mu_{\mathbf{l}}## and bounded, a reasonable hypothesis if we consider it the density of current, if...
Hello. I'm new to this forum. I'm starting a PhD – it's going to be a big long journey through the jungle that is CFD. I would like to arm myself with some tools before entering. The machete is Cartesian Tensors.
I know the rules regarding free suffix's and dummy suffixes, but I'm having...
Homework Statement
Find the scalar product of diracs delta function ##\delta(\bar{x})## and the bessel function ##J_0## in polar coordinates. I need to do this since I want the orthogonal projection of some function onto the Bessel function and this is a key step towards that solution. I only...
The following integral arises in the calculation of the new density of a non-uniform elastic medium under stress:
∫dx ρ(r,θ)δ(x+u(x)-x')
where ρ is a known mass density and u = ru_r+θu_θ a known vector function of spherical coordinates (r,θ) (no azimuthal dependence). How should the Dirac...
Lim as x approaches 4 of 1/x = 1/4
Given epsilon > 0, come up with a delta, d?Limits have been introduced. So far my instructor has had us make tables to see what value x was approaching. Although I don't understand exactly how limits are EVALUATED (different from looking at a chart & saying...
Homework Statement
a) ∫δ( x² cos x) dx with x from -∞ to +∞
b) Integral over C, with C = unit circle
∫C δ(α-α0) a ds
a =(1,1)
Homework Equations
a) δ(g(x)) =∑δ(x-xi)/|g'(xi)|
b)?
The Attempt at a Solution
a) g(x) = x² cosx
g'(x) = 2x cos x - x² sin x
xi = 0 and (2n+1)π/2...
Homework Statement
Use the epsilon delta definition to show that lim(x,y) -> (0,0) (x*y^3)/(x^2 + 2y^2) = 0
Homework Equations
sqrt(x^2) = |x| <= sqrt(x^2+y^2) ==> |x|/sqrt(x^2+y^2) <= 1 ==> |x|/(x^2+2y^2)?
The Attempt at a Solution
This limit is true IFF for all values of epsilon > 0, there...
Will epsilon delta test fail if curve changed direction within the +/- delta of the limit point?
Is there a scenario where no matter how small we pick delta to be, the frequency of the graph changing directions is always going to be a higher than delta's distance? In that scenario the limit...
Homework Statement
What can be special cases, in the study of electrical circuits, in which one can not apply wye delta transformation? Can someone graph them in some way?
Homework EquationsThe Attempt at a Solution
I am currently reading Modern Electrodynamics by Andrew Zangwill and came across a section listing some delta function identities (Section 1.5.5 page 15 equation 1.122 for those interested), and there is one identity that really confused me. He states:
\begin{align*}
\frac{\partial}{\partial...
Hello there !
I found this discussion http://physics.stackexchange.com/questions/155304/how-do-we-normalize-a-delta-function-position-space-wave-function about dirac notation and delta function .
The one that answers to the problem says that ##<a|a>=1## and ##<a|-a>=0## .
As far as i know:
1)...
Homework Statement
Compute the average value of the function:
f(x) = δ(x-1)*16x2sin(πx/2)*eiπx/(1+x)(2-x)
over the interval x ∈ [0, 8]. Note that δ(x) is the Dirac δ-function, and exp(iπ) = −1.
Homework Equations
∫ dx δ(x-y) f(x) = f(y)
The Attempt at a Solution
Average of f(x) = 1/8 ∫from...
I understand that there are vortices formed on the delta wing for moderate angles and low speeds. I observed recently 2 asymmetric vortices forming on a small delta wing model for a moderate angle and a low speed. But, why would there be asymmetriic flow over a delta wing? If the flow is...